Paradox in Solving y``+8y`+16y=64cosh4x

  • Thread starter Thread starter asdf1
  • Start date Start date
  • Tags Tags
    Paradox
Click For Summary
The discussion revolves around solving the differential equation y'' + 8y' + 16y = 64cosh4x. The initial approach using the particular solution yp = c1cosh4x + c2sinh4x leads to a paradox where the coefficients yield contradictory results. Suggestions include using the method of reduction of order and considering the associated homogeneous equation, specifically trying y = u(x)e^(-4x). It is noted that the method of undetermined coefficients is limited to specific function types, and corrections to the substitution are made. Ultimately, the correct approach involves recognizing that solutions to the homogeneous equation require adjustments, such as multiplying by x².
asdf1
Messages
734
Reaction score
0
for the following question:
y``+8y`+16y=64cosh4x

my problem:
suppose yp=c1cosh4x+c2sinh4x
then yp`=4c1sinh4x+4c2cosh4x
so yp``=16c1cosh4x+16sinh4x

so 16c1cosh4x+16sinh4x +8(4c1sinh4x+4c2cosh4x)+16(c1cosh4x+c2sinh4x)= (32c1+32c2)cosh4x+(32c2+32c1)sinh4x

which implies that (32c1+32c2)=0 and (32c2+32c1)=0 which is paradoxing!
does anybody know what went wrong?
 
Physics news on Phys.org
Try reduction of order. In using this method you can take just part of the solution to the associated homogeneous equation. I would try y = u\left( x \right)e^{ - 4x}. Any 'non-exponentials' eg polynomials in the complimentary solution get absorbed into u(x). Try the substitution I suggested and see if it leads anywhere.

Edit: The method of undetermined coefficients only works for a few types of functions.

Edit 2: I made an error in my suggested substitution. Fixed now.
 
Last edited:
Did you notice that e-4x and xe-4x are solutions to the homogeneous equation? Since 64 cosh 4x= 32(e4x+ e-4x) , you will have to multiply by x2. I would recommend trying y= Ae4x+ Bx2e-4x.
 
Last edited by a moderator:
hmmm... then what's wrong with my orignal assumptions? @@
 
Question: A clock's minute hand has length 4 and its hour hand has length 3. What is the distance between the tips at the moment when it is increasing most rapidly?(Putnam Exam Question) Answer: Making assumption that both the hands moves at constant angular velocities, the answer is ## \sqrt{7} .## But don't you think this assumption is somewhat doubtful and wrong?

Similar threads

  • · Replies 4 ·
Replies
4
Views
6K
Replies
5
Views
3K
  • · Replies 5 ·
Replies
5
Views
2K
  • · Replies 11 ·
Replies
11
Views
4K
  • · Replies 3 ·
Replies
3
Views
2K
  • · Replies 4 ·
Replies
4
Views
2K
Replies
2
Views
1K
Replies
2
Views
2K
  • · Replies 7 ·
Replies
7
Views
2K
Replies
2
Views
2K